Skip to main content
Mathematics LibreTexts

5.3: L'Hôpital's Rule

  • Page ID
    19051
  • \( \newcommand{\vecs}[1]{\overset { \scriptstyle \rightharpoonup} {\mathbf{#1}} } \) \( \newcommand{\vecd}[1]{\overset{-\!-\!\rightharpoonup}{\vphantom{a}\smash {#1}}} \)\(\newcommand{\id}{\mathrm{id}}\) \( \newcommand{\Span}{\mathrm{span}}\) \( \newcommand{\kernel}{\mathrm{null}\,}\) \( \newcommand{\range}{\mathrm{range}\,}\) \( \newcommand{\RealPart}{\mathrm{Re}}\) \( \newcommand{\ImaginaryPart}{\mathrm{Im}}\) \( \newcommand{\Argument}{\mathrm{Arg}}\) \( \newcommand{\norm}[1]{\| #1 \|}\) \( \newcommand{\inner}[2]{\langle #1, #2 \rangle}\) \( \newcommand{\Span}{\mathrm{span}}\) \(\newcommand{\id}{\mathrm{id}}\) \( \newcommand{\Span}{\mathrm{span}}\) \( \newcommand{\kernel}{\mathrm{null}\,}\) \( \newcommand{\range}{\mathrm{range}\,}\) \( \newcommand{\RealPart}{\mathrm{Re}}\) \( \newcommand{\ImaginaryPart}{\mathrm{Im}}\) \( \newcommand{\Argument}{\mathrm{Arg}}\) \( \newcommand{\norm}[1]{\| #1 \|}\) \( \newcommand{\inner}[2]{\langle #1, #2 \rangle}\) \( \newcommand{\Span}{\mathrm{span}}\)\(\newcommand{\AA}{\unicode[.8,0]{x212B}}\)

    We shall now prove a useful rule for resolving indeterminate limits. Below, \(G_{\neg p}\) denotes a deleted globe \(G_{\neg p}(\delta)\) in \(E^{1},\) or one about \(\pm \infty\) of the form \((a,+\infty)\) or \((-\infty, a).\) For one-sided limits, replace \(G_{\neg p}\) by its appropriate "half."

    Theorem \(\PageIndex{1}\) (L'Hôpital's rule)

    Let \(f, g : E^{1} \rightarrow E^{*}\) be differentiable on \(G_{\neg p}\), with \(g^{\prime} \neq 0\) there. If \(|f(x)|\) and \(|g(x)|\) tend both to \(+\infty,^{1}\) or both to \(0,\) as \(x \rightarrow p\) and if

    \[\lim _{x \rightarrow p} \frac{f^{\prime}(x)}{g^{\prime}(x)}=r \text { exists in } E^{*},\]

    then also

    \[\lim _{x \rightarrow p} \frac{f(x)}{g(x)}=r;\]

    similarly for \(x \rightarrow p^{+}\) or \(x \rightarrow p^{-}\).

    Proof

    It suffices to consider left and right limits. Both combined then yield the two-sided limit.

    First, let \(-\infty \leq p<+\infty\),

    \[\lim _{x \rightarrow p^{+}}|f(x)|=\lim _{x \rightarrow p^{+}}|g(x)|=+\infty \text { and } \lim _{x \rightarrow p^{+}} \frac{f^{\prime}(x)}{g^{\prime}(x)}=r\text { (finite)}.\]

    Then given \(\varepsilon>0,\) we can fix \(a>p\left(a \in G_{\neg p}\right)\) such that

    \[\left|\frac{f^{\prime}(x)}{g^{\prime}(x)}-r\right|<\varepsilon, \text { for all } x \text { in the interval }(p, a).\]

    Now apply Cauchy's law of the mean (§2, Theorem 2) to each interval \([x, a],\) \(p<x<a.\) This yields, for each such \(x,\) some \(q \in(x, a)\) with

    \[g^{\prime}(q)[f(x)-f(a)]=f^{\prime}(q)[g(x)-g(a)].\]

    As \(g^{\prime} \neq 0\) (by assumption), \(g(x) \neq g(a) \neq g(a)\) by Theorem 1, §2, so we may divide to obtain

    \[\frac{f(x)-f(a)}{g(x)-g(a)}=\frac{f^{\prime}(q)}{g^{\prime}(q)}, \quad \text { where } p<x<q<a.\]

    This combined with (1) yields

    \[\left|\frac{f(x)-f(a)}{g(x)-g(a)}-r\right|<\varepsilon,\]

    or, setting

    \[F(x)=\frac{1-f(a) / f(x)}{1-g(a) / g(x)},\]

    we have

    \[\left|\frac{f(x)}{g(x)} \cdot F(x)-r\right|<\varepsilon \text { for all } x \text { inside }(p, a).\]

    As \(|f(x)|\) and \(|g(x)| \rightarrow+\infty\) (by assumption), we have \(F(x) \rightarrow 1\) as \(x \rightarrow p^{+}\). Hence by rules for right limits, there is \(b \in(p, a)\) such that for all \(x \in(p, b)\), both \(|F(x)-1|<\varepsilon\) and \(F(x)>\frac{1}{2}\). (Why?) For such \(x\), formula (2) holds as well. Also,

    \[\frac{1}{|F(x)|}<2 \text { and }|r-r F(x)|=|r||1-F(x)|<|r| \varepsilon.\]

    Combining this with (2), we have for \(x \in(p, b)\)

    \[\begin{aligned}\left|\frac{f(x)}{g(x)}-r\right| &=\frac{1}{|F(x)|}\left|\frac{f(x)}{g(x)} F(x)-r F(x)\right| \\ &<2\left|\frac{f(x)}{g(x)} \cdot F(x)-r+r-r F(x)\right| \\ &<2 \varepsilon(1+|r|). \end{aligned}\]

    Thus, given \(\varepsilon>0,\) we found \(b>p\) such that

    \[\left|\frac{f(x)}{g(x)}-r\right|<2 \varepsilon(1+|r|), \quad x \in(p, b).\]

    As \(\varepsilon\) is arbitrary, we have \(\lim _{x \rightarrow p^{+}} \frac{f(x)}{g(x)}=r,\) as claimed.

    The case \(\lim _{x \rightarrow p^{+}} f(x)=\lim _{x \rightarrow p^{+}} g(x)=0\) is simpler. As before, we obtain

    \[\left|\frac{f(x)-f(a)}{g(x)-g(a)}-r\right|<\varepsilon.\]

    Here we may as well replace \(" a^{\prime \prime}\) by any \(y \in(p, a).\) Keeping \(y\) fixed, let \(x \rightarrow p^{+}\). Then \(f(x) \rightarrow 0\) and \(g(x) \rightarrow 0,\) so we get

    \[\left|\frac{f(y)}{g(y)}-r\right| \leq \varepsilon \text { for any } y \in(p, a).\]

    As \(\varepsilon\) is arbitrary, this implies \(\lim _{y \rightarrow p^{+}} \frac{f(y)}{g(y)}=r.\) Thus the case \(x \rightarrow p^{+}\) is settled for a finite \(r.\)

    The cases \(r=\pm \infty\) and \(x \rightarrow p^{-}\) are analogous, and we leave them to the reader. \(\quad \square\)

    Note 1. \(\lim \frac{f(x)}{g(x)}\) may exist even if \(\lim \frac{f^{\prime}(x)}{g^{\prime}(x)}\) does not. For example, take

    \[f(x)=x+\sin x \text { and } g(x)=x.\]

    Then

    \[\lim _{x \rightarrow+\infty} \frac{f(x)}{g(x)}=\lim _{x \rightarrow+\infty}\left(1+\frac{\sin x}{x}\right)=1 \quad(\text { why? }),\]

    but

    \[\frac{f^{\prime}(x)}{g^{\prime}(x)}=1+\cos x\]

    does not tend to any limit as \(x \rightarrow+\infty\).

    Note 2. The rule fails if the required assumptions are not satisfied, e.g., if \(g^{\prime}\) has zero values in each \(G_{\neg p};\) see Problem 4 below.

    Often it is useful to combine L'Hôpital's rule with some known limit formulas, such as

    \[\lim _{z \rightarrow 0}(1+z)^{1 / z}=e \text { or } \lim _{x \rightarrow 0} \frac{x}{\sin x}=1 \text { (see §1, Problem 8).}\]

    Examples

    (a) \(\lim _{x \rightarrow+\infty} \frac{\ln x}{x}=\lim _{x \rightarrow+\infty} \frac{(\ln x)^{\prime}}{1}=\lim _{x \rightarrow+\infty} \frac{1}{x}=0.\)

    (b) \(\lim _{x \rightarrow 0} \frac{\ln (1+x)}{x}=\lim _{x \rightarrow 0} \frac{1 /(1+x)}{1}=1.\)

    (c) \(\lim _{x \rightarrow 0} \frac{x-\sin x}{x^{3}}=\lim _{x \rightarrow 0} \frac{1-\cos x}{3 x^{2}}=\lim _{x \rightarrow 0} \frac{\sin x}{6 x}=\frac{1}{6} \lim _{x \rightarrow 0} \frac{\sin x}{x}=\frac{1}{6}.\)

    (Here we had to apply L'Hôpital's rule repeatedly.)

    (d) Consider

    \[\lim _{x \rightarrow 0^{+}} \frac{e^{-1 / x}}{x}.\]

    Taking derivatives (even \(n\) times), one gets

    \[\lim _{x \rightarrow 0^{+}} \frac{e^{-1 / x}}{n ! x^{n+1}}, \quad n=1,2,3, \ldots \text { (always indeterminate!).}\]

    Thus the rule gives no results. In this case, however, a simple device helps (see Problem 5 below).

    (e) \(\lim _{n \rightarrow \infty} n^{1 / n}\) does not have the form \(\frac{0}{0}\) or \(\frac{\infty}{\infty},\) so the rule does not apply directly. Instead we compute

    \[\lim _{n \rightarrow \infty} \ln n^{1 / n}=\lim _{n \rightarrow \infty} \frac{\ln n}{n}=0 \text { (Example (a))}.\]

    Hence

    \[n^{1 / n}=\exp \left(\ln n^{1 / n}\right) \rightarrow \exp (0)=e^{0}=1\]

    by the continuity of exponential functions. The answer is then 1.


    This page titled 5.3: L'Hôpital's Rule is shared under a CC BY 3.0 license and was authored, remixed, and/or curated by Elias Zakon (The Trilla Group (support by Saylor Foundation)) via source content that was edited to the style and standards of the LibreTexts platform; a detailed edit history is available upon request.